Changes to the ACT in 2025

In response to the shorter, digital version of the SAT, the ACT announced that they too will be modifying their test.  The two major problems students have with the ACT are (1) it takes too long to complete a full test and (2) they often do not have enough time to answer all the questions.  These changes will tackle these concerns, making the ACT much more competitive with the SAT. 

Here is what we know for sure about the new ACT:

  • The ACT will be about an hour shorter (2 hours of testing instead of 3) thanks to making the sections shorter and making the Science section optional.
  • Composite scores (still given from 1-36 points) will be an average of just the English, Math, and Reading sections.
  • Students will have more time per question.
  • The ACT will continue to offer both paper and digital versions of the test.
  • According to ACT, the changes “will roll out starting with National online testing in spring 2025 and then for school-day testing in spring 2026.”

What do we need to find out about the new ACT?

  • What is the precise design of the updated ACT test?  If it is consistent with the format of the test used in a research study in June of this year, it will likely have these parameters:
    • English:  35 minutes, 50 questions, 5 or 10 questions on each passage
    • Math:  50 minutes, 45 questions
    • Reading:  40 minutes, 36 questions, 9 questions on each passage
    • Optional Science:  45 minutes, 36 questions
  • What sort of ACT practice materials will be provided?  When the SAT changed its format, it made four brand new SAT practice tests available to students prior to the first administered test.  Since the ACT does not routinely make significant updates to its structure, it is unclear how much material they will make available prior to the first test. 
  • When will the switch for the national paper ACTs take place?  The announcement from ACT does not make this clear, only referring to the national online testing.  The switch to the newer format on the national paper tests won’t happen before February of 2025, but it could happen sometime during 2025 or even early 2026.  We will have to wait and see. 
  • How will colleges use ACT scores?  Our best guess is that the ACT composite score with just the English, Math, and Reading sections will be sufficient.  However, students who want to demonstrate their competency for science-related majors may be well-served by doing the ACT science. 

What should students do about the new ACT? 

  • Rising seniors in the class of 2025 do not need to do anything.  These changes will go into effect after college applications have been submitted. 
  • Rising juniors in the class of 2026 may want to try the paper ACT before it goes away; they can also try the updated ACT when it is convenient do so.  When the SAT changed to its digital format, many students took advantage of trying both the paper SAT before it went away and the newer digital SAT.  It is highly likely that colleges will accept scores from either the newer ACT or the current ACT.  When students prepare for the new ACT, they will likely be able to use existing practice materials; they can try working through them at a slightly slower pace to reflect the more generous time constraints on the new ACT. 
  • Underclassmen—classes of 2027 and later—may want to wait to begin focused preparation on the ACT until there is more information available.  If a highly motivated sophomore wants to take the longer ACT before it goes away, they might go ahead and give it a try. 

Please reach out to us with questions you may have about these major changes to the ACT and how they affect your testing plans.    

Digital SAT Reading Practice: Main Idea Questions


1. The following text is adapted from Robert Louis Stevenson’s 1888 book The Black Arrow—A Tale of the Two Roses:

One afternoon, the bell upon Tunstall Moat House was heard ringing at an unaccustomed hour. Far and near, in the forest and in the fields along the river, people began to desert their labors and hurry towards the sound. By the bridge, there was a stone cross upon a knoll, and here a group had collected—half a dozen women and one tall fellow in a russet smock—discussing what the bell betided. An express had gone through the hamlet an hour before, and drunk a pot of ale in the saddle, not daring to dismount for the hurry of his errand; but he had been ignorant himself of what was forward, and only bore sealed letters to Sir Oliver Oates who kept the Moat House in the master’s absence.

What is the main idea of the text?
A. The church bell has called people together to learn happy news from a stranger.
B. A messenger has brought news that is yet unknown, yet important enough for all the local people to be called together.
C. The bell upon Tunstall Moat House is broken, causing it to ring at an unknown time. A group of people gather together to discuss the issue and the unknown solution.
D. A sealed letter has arrived, causing the messenger to ring the bell at Tunstall Moat House and bring together all the people.

2. The following text is adapted from Arthur Morrison’s novel The Dorrington Deed-Box:

Dorrington looked grave. “It’s really extraordinary,” he said, “most extraordinary; and it isn’t often that I call a thing extraordinary, with my experience. But it’s plain something must be done—something to gain time at any rate. We’re in the dark at present, of course, and I expect I shall have to fish about a little before I get at anything to go on. In the meantime, I think you must disappear as artfully as we can manage it.” He sat silent for a little while, thoughtfully tapping his forehead with his finger-tips. “I wonder,” he said presently, “whether or not those Italian fellows on the steamer are in it or not. I suppose you haven’t made yourself known anywhere, have you?” “Nowhere. As you know, you’ve been with me all the time”

What is the main idea of the text?
A. The man Dorrington is speaking to has an extraordinary talent which has lead to him being chased by the Italians.
B. Dorrington is in trouble and looking to gain time in which to solve a problem he is facing.
C. The man to whom Dorrington is speaking is in some sort of mysterious trouble and Dorrington, in order to help, suggests that he disappear.
D. Two fishing buddies hatch a plan to excel in their hobby, despite the odds being against them.

3. The following text is adapted from Baroness Orczy’s The League of the Scarlet Pimpernel. In it, Madeleine Lannoy is promised help in finding her lost child.

Dante, in his wild imaginations of hell and of purgatory and fleeting glimpses of paradise, never put before us such a picture of a soul that was lost and found heaven, after a cycle of despair. Nor could Madeleine Lannoy ever explain her feelings at that moment, even to herself. To begin with, she could not quite grasp the reality of this ray of hope, which came to her at the darkest hour of her misery. She stared at the man before her as she would an ethereal vision; she fell on her knees and buried her face in her hands.

What is the main idea of the text?
A. Dante’s picture of hell and redemption shows us what Madeleine Lannoy has suffered in her despair and grief for her child.
B. The hope that Madeleine Lannoy feels is such a transformation for her that not even Dante’s literary move from hell to heaven could compare with the change.
C. A man appears before Madeleine Lannoy, and, not knowing if he is from heaven or hell, she sinks to her knees and trembles in fear.
D. Madeleine Lannoy, in her agony, despairs at ever finding her lost child, and compares her life to Dante’s inferno.

4. The following text is adapted from John Buchan’s novel Mr. Standfast:

He leaned forward and tapped me on the knee. “I reverence the British Intelligence Service. Flies don’t settle on it to any considerable extent. It’s got a mighty fine mesh, but there’s one hole in that mesh, and it’s our job to mend it. There’s a high-powered brain in the game against us. I struck it a couple of years ago when I was hunting Dumba and Albert, and I thought it was in New York, but it wasn’t. I struck its working again at home last year and located its head office in Europe. So I tried Switzerland and Holland, but only bits of it were there. The center of the web where the old spider sits is right here in England, and for six months I’ve been shadowing that spider.”

What is the main idea of the text?
A. There is an issue with the netting around the characters which is letting in spiders and other creatures.
B. The British Intelligence Service, and the speaker specifically, is struggling to update its files on issues around the world.
C. The speaker is trying to follow a covert operative who is continually moving through Europe, with his headquarters in England.
D. The speaker is following and learning of an unknown threat which is centered in England and has reach in countries around the world.

5. The following text is adapted from James Fenimore Cooper’s 1826 novel The Last of the Mohicans:

The route taken by Hawkeye lay across those sandy plains, relived by occasional valleys and swells of land, which had been traversed by their party on the morning of the same day, with the baffled Magua for their guide. The sun had now fallen low toward the distant mountains; and as their journey lay through the interminable forest, the heat was no longer oppressive. Their progress, in consequence, was proportionate; and long before the twilight gathered about them, they had made good many toilsome miles on their return.

What is the main idea of the text?
A. A party of people undertakes a journey across many different terrains and finds that they can progress at a decent rate.
B. Hawkeye alone travels across plains, valleys, and swells in search of a party of people who have become lost.
C. Let by Magua, Hawkeye finds a route through the wilderness as a larger party of people make good time in their journey.
D. Surrounded by wilderness, a group of people, led by their guide, struggles to reach the mountains across plains and valleys, and through a dense forest.

6. The following text is adapted from Sir Walter Scott’s 1819 novel Ivanhoe

“Cedric is not her father,” replied the Prior, “and is but of remote relation: she is descended from higher blood than even he pretends to, and is but distantly connected with him by birth. Her guardian, however, he is, self-constituted as I believe; but his ward is as dear to him as if she were his own child.”

What is the main idea of the text?
A. Cedric and the girl are not related to one another even though they love each other.
B.  Cedric’s daughter is from a very noble family on her mother’s side and this separates her from him and his low birth.
C. Cedric and his ward are only distantly related. But despite this and their difference in nobility, Cedric loves her as a daughter.
D. Cedric’s ward is not his daughter and will not let her guardian forget that she is of nobler birth than he.

7. The following text is adapted from George Eliot’s 1872 novel Middlemarch:

Dorothea trembled while she read this letter; then she fell on her knees, buried her face, and sobbed. She could not pray: under the rush of solemn emotion in which thoughts became vague and images floated uncertainly, she could but cast herself, with a childlike sense of reclining, in the lap of a divine consciousness which sustained her own. She remained in that attitude till it was time to dress for dinner.

What is the main idea of the text?
A. Dorthea has read an upsetting letter which has caused her to fall to her knees and seek comfort in the divine.
B. While reading a letter, Dorthea was overcome by emotion and loses her consciousness to grief and fear.
C. Overcome by the contents of a letter Dorthea kneels in prayer of thanksgiving that she has been sustained through terrible news.
D. Having fallen ill, Dorthea seeks comfort in her letters and in divine providence as she drifts through her consciousness.

8. The following text is adapted from William Makepeace Thackeray’s 1848 novel Vanity Fair. In it the author discuss a woman who, despite already having a large house and many servants, desires to move up in society.

And so—guiltless very likely—she was writhing and pushing onward towards what they call “a position in society,” and the servants were pointing at her as lost and ruined. So you see Molly, the housemaid, of a morning, watching a spider in the doorpost lay his thread and laboriously crawl up it, until, tired of the sport, she raises her broom and sweeps away the thread and the artificer.

What is the main idea of the text?
A. Molly, lost in dreams of a better position in society, slacks on the job, choosing to watch a spider instead of cleaning.
B. The mistress of the house, in following her social aspirations, has lost the respect of her servants.
C. Molly, instead of doing her work, spends her time watching spiders and mocking her employer.
D. Desiring a spot in society, the mistress of the house allows her home to fall into disrepair and uncleanliness.

9. The following text is adapted from Mark Twain’s 1882 novel The Prince and the Pauper:

“He must be the prince!  Will any be in all the land maintain there can be two, not of one blood and birth, so marvellously twinned?  And even were it so, ’twere yet a stranger miracle that chance should cast the one into the other’s place. Nay, ’tis folly, folly, folly!”

Presently he said— “Now were he impostor and called himself prince, look you that would be natural; that would be reasonable.  But lived ever an impostor yet, who, being called prince by the king, prince by the court, prince by all, denied his dignity and pleaded against his exaltation?  No!  By the soul of St. Swithin, no!  This is the true prince, gone mad!”

What is the main idea of the text?
A. The boy who is called the prince is not the prince, but rather an imposter.
B. The prince must have gone mad—no other explanation makes sense to the speaker.
C. The speaker, the king, and the listener are all trying to determine if the boy in question is the prince.
D. No one knows if the boy in question is the prince or an imposter.

10. The following text is adapted from Charlotte Bronte’s 1849 novel Shirley: I did not find it easy to sketch Mr. Yorke’s person, but it is more difficult to indicate his mind. If you expect to be treated to a Perfection, reader, or even to a benevolent, philanthropic old gentleman in him, you are mistaken. He has spoken with some sense and with some good feeling to Mr. Moore, but you are not thence to conclude that he always spoke and thought justly and kindly.

What is the main idea of the text?
A. The author struggles to describe Mr. Yorke’s personality and therefore gives up, leaving the reader with no clear picture of the man.
B. Mr. Yorke is best know to the reader for how kindly he treated Mr. Moore. It therefore is best for the reader to understand Mr. Yorke as a kind man.
C. The author feels he cannot well describe Mr. Yorke’s personality, but attempts to make sure that the reader knows that Yorke is not a kindly old gentlemen despite his actions toward Mr. Moore.
D. Mr. Yorke, despite being known as a kind man for his actions toward Mr. Moore, is actually more than that. He is generous with his money, just, and kind toward all around him.

  1. B. The passage does not explain what is in the letter that the messenger has brought. However, the people gathered have connected the letter to the bell which is ringing to gather the people together. This best fits with answer option B. Answer option A is incorrect because it claims to know that the news is happy. Answer option C is incorrect as there is not evidence that the bell is broken. Answer option D is incorrect as there is no evidence that the messenger himself is ringing the bell.
  2. C. In the passage, Dorrington is commenting on an “extraordinary” situation that they don’t know much about and that necessitates the other character’s disappearance. We can surmise that the other character must be in some trouble and since Dorrington is going to “fish about a little” for him he must be trying to help. This best fits with answer B. Answer A is incorrect as there is no evidence of an extraordinary talent or the Italians chasing him. Answer B is incorrect as it seems that the other man, not Dorrington, is in trouble. Answer D is incorrect as it misreads the fishing comment and other parts of the passage as literal when they are figurative.
  3. B. The first sentence points out that even Dante’s description of hell and heaven “never put before us” an example like that of Madeleine who has been pulled from the hell of losing her child and given a ray of hope. This best fits answer B. Answer A is incorrect as the passage says that Dante’s work is not enough to describe the situation. Answer C is incorrect as the man before her is not actually an ethereal vision, she merely looks at him like one. Answer D is incorrect as Madeleine now feels some hope, not despair.
  4. D. The speaker talks about how there is a hole in the mesh and it’s his job to fix it. In trying to fix it he has traveled the world looking for the “spider” and discovered that while parts of it are in different places around the world, the spider is in England and he has been following it. This spider is a threat that is unknown. Which makes option D the best answer. Answer A is wrong as the spider and mesh are literal, not figurative. Answer B is wrong as it misunderstands the reference to the files. Answer C is incorrect as there is no evidence that the spider is a single covert operative.
  5. A. The main idea of the text centers around the discussion of the journey being made by the group. The journey’s path is described and then the passage ends with a note that due to the relative cool the group managed to progress many miles. This best fits with answer A. Answer B is incorrect as the passage does not focus on Hawkeye’s individual journey nor does it indicate that the people are lost. Answer C is incorrect as Magua is not Hawkeye’s guide. Answer D is incorrect as we do not know if the group is trying to reach the mountains or another destination.
  6. C. Cedric is not the girl’s father as is plainly stated. He is, however, a “remote” relation, meaning they are distantly related. In addition, Cedric has taken on the role of guardian and loves the girl as his own child. There is no evidence that her high birth has caused any issues between them. This makes option C the best option. Option A is incorrect as it claims they are not related at all. Option B is incorrect as the girl is not Cedric’s daughter. Option D is incorrect as there is no evidence that the girl holds his birth against him.
  7. A. Notice that Dorthea trembles even as she reads the letter and, when finished, seems to fall apart. It would seem that the contents of the letter have caused this reaction. This excludes option D: it is not a physical malady that has cause Dorthea’s reaction. We have no evidence that Dorthea is specifically fearful, or even that she feels grief, which eliminates option B. We know that Dorthea does not pray, which eliminates option C. This only leaves option A. Dorthea cannot pray but she does cast herself “into the lap of a divine consciousness which sustains her own”. So, while she does not pray, she does seek comfort in the divine.
  8. B. The woman discussed in the first sentence must be the mistress of the house, given the background information shared before the passage. She is described as wanting “a position in society” which has caused the servants to disrespect her, pointing, and calling her ruined. This leads to her servants, like Molly, not behaving as they should under her employ. This best fits answer option B. Answer option A is incorrect as it is not Molly who desires a better position. Answer option C is incorrect as this is part of the passage, but not the main idea. Option D is incorrect as there is no evidence that the house is in disrepair.
  9. B. In the passage, the speaker insists that the boy must be the prince. Nothing else makes sense to him. He does not think that there could be another person who so resembles the prince, nor does he think that an imposter would ever change his mind and begin to say that he isn’t  the prince. He points out that the king himself accepts that this boy is his son and concludes that the boy must be the prince and must be going mad to try to say he isn’t himself. This makes option B the best answer. Answer A is incorrect as the speaker does not think there is an imposter. Answer C is incorrect as the king and speaker consider it settled that the boy is not an imposter. Answer D is incorrect as the question in the passage is treated as settled, not as something that is unknown.
  10. C. The author at the beginning admits that he struggles to explain Mr. Yorke’s mind (or personality) to the reader. He does, however, make sure that the reader does not see Mr. Yorke as a “benevolent, philanthropic old gentleman” and he warns the reader not to assume that his kind actions to Mr. Moore are representative of how he acts all the time. This best fits with answer C and makes the other options incorrect.

Digital SAT Practice: Writing/Verb Choice Questions #2

1. The mother of Dr. Haddock was Abigail Webster, a favorite sister of Ezekiel and Daniel Webster, who, with Sarah, ______ the only children of the Hon. Ebenezer Webster by his second wife, Abigail Eastman, who survived her husband and all her daughters.

Which choice completes the text so that it conforms to the conventions of standard English?
A. was
B. were
C. are
D. is

2. Mr. Layard in excavating beneath the great pyramid at Nimroud, had penetrated a mass of masonry, within which he had discovered the tomb and statue of Sardanapalus, accompanied by full annals of the monarch’s reign engraved on the walls! He had also found tablets of all sorts, all of them being historical, but the crowning discovery he _____________________

Which choice completes the text so that it conforms to the conventions of standard English?
A. had yet to describe.
B. still has to describe.
C. yet describes.
D. hadn’t yet to describe.

3. It is indeed a painful or rather a terrible condition in which Heine now is and _________ for the past year; though the paralysis has made no progress, it has at least experienced no alleviation. He has now lain near two years in bed, and during that time has not seen a tree nor a speck of the blue sky.

Which choice completes the text so that it conforms to the conventions of standard English?
A. were
B. was
C. has been
D. will be

4. Dr. Otto Zirckel has just published at Berlin a volume called “Sketches from and concerning the United States,” which has some curious peculiarities to the eyes of an American. It is intended as a guide for Germans who wish either _________________ to this country or to send their money here for investment.

Which choice completes the text so that it conforms to the conventions of standard English?
A. to have emigrated
B. to emigrate
C. emigration
D. emigrating

5. The omissions by Mr. Sparks—sometimes from carelessness, sometimes from ignorance, and sometimes from an indisposition to revive memories of old feuds, or to cover with disgrace names which should be dishonored—and his occasional verbal alteration of Washington’s letter, _________ that general satisfaction with which his edition of Washington would otherwise be regarded.

Which choice completes the text so that it conforms to the conventions of standard English?
A. prevents
B. has prevented
C. preventing
D. prevent

6. Bauer treats the political and religious parties of modern Germany with the same scornful satire and destructive analysis which __________ in his theological writings.

Which choice completes the text so that it conforms to the conventions of standard English?
A. appear
B. will appear
C. has appeared
D. appears

7. Dr. Laing, one of those restless English travelers who have printed books about the United States, _____ now a prominent personage in Australia, where he has been elected a member of the newly instituted Legislature, for the city of Sidney.

Which choice completes the text so that it conforms to the conventions of standard English?

A. was
B. is
C. are
D. were

8. The first two volumes of Oehlenschlager’s Lebens Erinnerungen have appeared at Vienna, and ___________ more observation than anything else in the late movements in the German literature.

Which choice completes the text so that it conforms to the conventions of standard English?
A. attract
B. attracts
C. will attract
D. attracted

9. The character of Sir Roger de Coverley is a creation which, in its way, has never been surpassed; never perhaps __________ except by the Vicar of Wakefield.

Which choice completes the text so that it conforms to the conventions of standard English?

A. equals
B. equaling
C. equal to
D. equaled

10. When the public understands how perfectly Parodi identifies herself with the emotions and passions she has to portray,—when they appreciate the immense variety of intonations with which she illustrates her characters, and the earnestness and intensity with which she _________her whole nature into all she does—then she will be hailed as the greatest artist ever on this continent, and one of the greatest in the world.

Which choice completes the text so that it conforms to the conventions of standard English?

A. throw
B. throws
C. has thrown
D. can throw

  1. A. The subject of the verb is both Abigail Webster and her Sarah. This makes our subject plural. In addition, this passage is in the past tense, which makes B the best option since it is plural and in the past tense.
  2. A. The passage is primarily written in the past perfect, the tense which uses a helping verb like “has” or “had” along with the past participle of the verb. To continue this style, we should use answer option A. Answer option B is not in the past perfect as it uses the word “still”. Answer C does not use a helping verb. Answer D incorrectly uses the word not.
  3. C. Heine is currently in this bad condition and for the past year and up until this moment he “has been” in this condition. Option A is plural where our subject is singular. Option B implies that he was in this condition in the past but is not still in the condition, and option D put this into the future despite it being in the “past year”. This makes option C the best answer.
  4. B. Look for parallel structure within the sentence. The Germans may wish to do two things. The second thing is “to send”. We therefore need to match that with our choice “to emigrate”. This makes option B correct and all other options incorrect.
  5. D. Be careful in identifying the subject of the verb. Ask yourself, “what is it that is going to prevent the satisfaction?” In this case, it is the omissions all the way at the beginning of the passage. “Omissions” is plural and so the correct verb choice would be “prevent”. This makes option D correct and the others incorrect.
  6. A. Be careful in identifying the subject of the verb. Ask yourself, “what is it that we’re talking about appears in his theological writings?” It is the scornful satire and destructive analysis. This means that the subject of our verb is plural. Our verb must work with a plural subject eliminating options C and D. Option B is incorrectly in the future tense. This just leaves option A as the correct answer.
  7. B. Be careful in identifying the subject of the verb. Ask yourself, “who is a prominent personage?” it can’t be “the United States”. Because the U.S. is neither a person nor in Australia. The person is Dr. Laing. This means that we must pick a singular verb, eliminating options C and D. Since it is “now” we must pick the present tense option B and not the past tense option A.
  8. D. These first two volumes have done two things they have “appeared” and “attracted”. The second verb must match the tense of the first making option D correct and the other options incorrect.
  9. D. Make sure to maintain parallel structure. Sir Roger de Coverley has never been “surpassed” so then we must say he has also never been “equaled”. This makes option D correct and the others incorrect.
  10. B. Make sure to maintain parallel structure. People appreciate two things about Parodi: how she “illustrates” her character and how she “throws” her whole nature. Both verbs need to be in the present tense making option B correct and the other options incorrect.

DSAT Reading Words in Context Practice Questions #2

1. Up to this moment the young earl had stood still, as if spell-bound; but being now convinced that the spirit had fled, he pressed forward, and, ere many seconds, emerged from the brake. The full moon was rising as he ______________ and illuminating the glades and vistas, and the calmness and beauty of all around seemed at total variance with the fearful vision he had just witnessed.

Which of the following choices best completes the text with the most logical and precise phrase?
A. went from,
B. issued forth,
C. cropped up,
D. came across,

2. They walked on in silence, for the earl could not help dwelling upon the vision he had witnessed, and his companion appeared equally ____________ In this sort they descended the hill near Henry the Eighth’s Gate, and entered Thames Street.

Which of the following choices best completes the text with the most logical and precise word?
A. unconcerned.
B. worried.
C. abstracted.
D. alive.

3. Turning off on the left into the lower road, ________ around the north of the castle, and following the course of the river to Datchet, by which it was understood the royal cavalcade would make its approach, the procession arrived at an open space by the side of the river, where it came to a halt.

Which of the following choices best completes the text with the most logical and precise word?
A. dressing
B. skirting
C. avoiding
D. leaving

4. Presently the sound of trumpets smote his ear, and a numerous and splendid retinue was seen advancing, consisting of nobles, knights, esquires, and gentlemen, ranged according to their degrees, and all sumptuously ___________ in cloths of gold and silver, and velvets of various colors, richly embroidered.

Which of the following choices best completes the text with the most logical and precise word?
A. worn
B. addressed
C. redressed
D. appareled

5. The person who thus spoke then stepped forward, and threw a glance so full of significance at Anne Boleyn that she did not care to dispute the order, but, on the contrary, laughingly ______________ it.

Which of the following choices best completes the text with the most logical and precise word or phrase?
A. railed to
B. railed against
C. acquiesced against
D. acquiesced to

6. The favor in which he stood with his royal master procured him admittance to his presence at all hours and at all seasons, and his influence, though seldom exerted, was very great. He was especially serviceable in turning aside the edge of the king’s displeasure, and more frequently exerted himself to _______ the storm than to raise it.

Which of the following choices best completes the text with the most logical and precise word?
A. allay
B. allow
C. align
D. stoke

7. He took her hand, and led her to the upper part of the chamber, where two chairs of state were set beneath a canopy of crimson velvet embroidered with the royal arms, and placed her in the seat hitherto allotted to the previous queen. A smile of triumph irradiated Anne’s lovely countenance at this ________________ nor was her satisfaction diminished as Henry turned to address the assemblage.

Which of the following choices best completes the text with the most logical and precise phrase?
A. show of force,
B. mark of distinction,
C. level of crime,
D. morbid scene,

8. This ghostly rider wore the antlered helmet described by Surrey, and seemed to be habited in a garb of deer-skins. Before him flew a large owl, and a couple of great black dogs ran beside him. Staring in _____________ wonder at the sight, the two youths watched the mysterious being scour a glade brightly illumined by the moon, until, reaching the pales marking the confines of the Home Park, he leaped them and disappeared.

Which of the following choices best completes the text with the most logical and precise word?
A. speechless
B. unspeakable
C. amazed
D. lonely

9. Before they reached the hill, at the end of the long avenue, a heavy thunderstorm came on, and the lightning, playing among the trees, seemed to reveal a thousand fantastic forms to their half-blinded gaze. Presently the rain began to descend in torrents, and compelled them to ___________beneath a large beech-tree.

Which of the following choices best completes the text with the most logical and precise phrase?
A. abide beneath
B. stay wet
C. align themselves
D. take refuge

10. A verdant path, partly beneath the trees, and partly on the side of the lake, led Wolsey to the forester’s hut. Constructed of wood and clay, with a thatched roof, green with moss, and half overgrown with ivy, the little building was in __________ keeping with the surrounding scenery.

Which of the following choices best completes the text with the most logical and precise word?
A. admiring
B. admirable
C. poor
D. abominable

  1. B. The young earl “emerged from the brake”. We are looking for a word that would fit the context of the sentence and describe his emerging. Answer option A does not fit the structure of the sentence as there is no further description of where he went from. Answer C implies something slowly sneaking up over time, which is not the proper in this context. Answer D does not mean “emerging” and is therefore incorrect. This only leaves answer option B, that he issued forth (meaning “emerged”).
  2. C. The earl is dwelling on the vision, not on his companion or on making conversation. He is distracted by what they have seen. His companion, then, is equally distracted. Abstracted is a synonym for distracted, making option C the best answer. Answer A is the opposite of the correct answer. Answer B is incorrect as there is no evidence the earl is worried, only that he is distracted and thinking about the vision. There is no discussion of them feeling or being alive or dead, making answer option D incorrect.
  3. B. Given that the blank is followed by the word “around” we must choose an answer that can be done “around the north of the castle”. A procession could not dress around a castle, nor could it avoid or leave around it. This leaves answer B as the correct answer. To “skirt around” means to go around the edges of something.
  4. D.  The context is describing what the gentlemen are wearing. This makes options B and C incorrect as they are both words that look similar to the word “dressed” but have different meanings. “Worn” does not fit the context of the sentence. This leaves answer option D. The men are “sumptuously appareled in cloths”. Appareled is a synonym for “dressed”.
  5. D. We see in the first part of the sentence that Anne is not going to dispute (go against) the order. The author then sets up a contrasting statement by saying “but, on the contrary”. We therefore need to pick an answer that means the opposite of “dispute”. To acquiesce means to give in. The appropriate preposition for acquiesce is “to”. This makes option D correct. You cannot acquiesce against something, making option C incorrect and options A and B would not show contrast with “dispute” since “to rail against” means to speak aggressively against something.
  6. A. We learn in the first sentence that this man is favored by the king and has great influence over the king. The second sentence shows us how the man uses that influence to turn “aside the edge of the king’s displeasure.” The final phrase sets up a contrast. Rather than raising the storm of anger he is more likely to subdue it. This is the meaning of the word “allay” (related to the word alleviate) which makes answer option A correct. Answer options B and D mean close to the opposite of the author’s intended meaning. Answer option C is incorrect as “align” is not something that can be done with storms or anger.
  7. B. Anne is taken to sit in the seat of the queen this is a good thing for Anne and elicits from her a “smile of triumph”, making options C and D incorrect. Option A is incorrect as there is no evidence that Henry has shown any force toward her. Instead, being sat in the queen’s chair is a “mark of distinction” setting her apart from anyone else who might be watching.
  8. A. The boys are watching a ghostly rider, which makes their wonder understandable. They are together and so option D doesn’t fit. Their wonder isn’t unspeakable, a word for things so horrible they can’t be said. “Amazed wonder” would be redundant. This leaves option A. They are speechless with wonder.
  9. D. Given the storm, the people need to find a safe place. They “take refuge” under a tree to escape the storm. They are not living beneath the tree, making option A incorrect. They would not want to stay wet, but rather, dry making option B incorrect. They are not aligning themselves with anything, making option C incorrect.
  10. B. We see that the house is beneath trees, and on the side of the lake. It is made of natural materials and therefore matches the scenery. This makes options C and D incorrect as they would imply that the house does not match the scenery. The house is not admiring, making option A incorrect.


    If you would like to expand your vocabulary by reading of the story of Anne, Henry, and ghostly riders, the whole book is available for free through Project Gutenberg:
    https://www.gutenberg.org/cache/epub/2866/pg2866-images.html

In a Major Reversal, Harvard Will Again Require ACT and SAT Scores

Following a wave of other prestigious colleges–Brown, Dartmouth, Yale– that have changed from test optional to test required, Harvard University announced today that they will again require standardized test scores for undergraduate admissions:

https://www.thecrimson.com/article/2024/4/11/harvard-sat-act-admissions-requirement/

This shift is especially notable because Harvard had previously stated that they would remain test optional up through applicants for the Harvard graduating class of 2030.

Why this change? First, “Harvard has found that SAT and ACT scores are  the best predictors of Harvard grades.” This comes as no surprise in light of increasing grade inflation at U.S. high schools. According to Inside Higher Ed, “the proportion of students with A averages (including A-minus and A-plus) increased from 38.9 percent of the graduating class of 1998 to 47 percent of the graduating class of 2016.” If most applicants are getting A’s, colleges need another way to differentiate among applicants–hence, the need for standardized test scores.

Second, many students have mistakenly been told that unless they get ACT scores in the high 30s or SAT scores above 1500, they have no business applying to selective schools. Harvard wants to encourage students to submit scores who fall below this range, sharing that “in the last year that Harvard required testing, the range (10th percentile to 90th percentile) of SAT scores for enrolling students was 670 to 790 for Evidence-Based Reading and Writing and 680 to 800 for Math.  The range (10th percentile to 90th percentile) of ACT Composite scores was 31 to 36.” Harvard applicants who score in the in the high 1300s on the SAT and low 30s on the ACT should definitely considering submitting their scores based on this data.

Most college admissions officials to whom I have spoken have said that students are typically well-advised to submit their test scores if they are at least at the 25th percentile for admitted students. Now that standardized testing is widely available, unlike during the Covid pandemic, applicants should be aware that if they decline to submit their test scores to a college, they are effectively signaling that their scores are less than the 25th percentile for admitted students. Students can find the 25th percentile scores for colleges by searching the Big Future website.

The bottom line–in order to differentiate yourself in an increasingly competitive admissions environment, ACT and SAT scores should not longer be considered optional.

–Brian W. Stewart

Digital SAT Practice Questions: Command of Evidence

1. The following is an excerpt from Essays by Ralph Waldo Emerson:

The farmer imagines power and place are fine things. But the President has paid dear for his White House.  It has commonly cost him all his peace, and the best of his many attributes. To preserve for a short time so conspicuous an appearance before the world, he is content to eat dust before the real masters who stand erect behind the throne. Or, do men desire the more substantial and permanent grandeur of genius? Neither has this an immunity. He who by force of will or of thought, is great, and overlooks thousands, has the charges of that eminence. With every influx of light comes new danger.

Based on the passage, which of the following would best describe the author’s attitude toward power?
A. Power is something to which all men strive and few men attain. Power once found will always corrupt.
B. While men see power as something laudable and to be sought after, they forget that it comes at a steep price.
C. Power should be avoided by all men who can stay away from it. Each man should be content where he is.
D. Power is all well and fine for the president and similar lofty persons, but is not suitable for the common farmer.

2.  The following is an excerpt from Essays by Ralph Waldo Emerson:

The scholar of the first age received into him the world around; brooded thereon; gave it the new arrangement of his own mind, and uttered it again. It came into him life; it went out from him truth. It came to him short-lived actions; it went out from him immortal thoughts. It came to him business; it went from him poetry. It was dead fact; now, it is quick thought. It can stand, and it can go. It now endures, it now flies, it now inspires. Precisely in proportion to the depth of mind from which it issued, so high does it soar, so long does it sing.

Based on the passage, what is the “it” referred to throughout the paragraph?
A. Poetry
B. Music
C. The world
D. Actions

3. The following is an excerpt from Essays by Ralph Waldo Emerson:

Society everywhere is in conspiracy against the manhood of every one of its members. Society is a joint-stock company, in which the members agree, for the better securing of his bread to each shareholder, to surrender the liberty and culture of the eater. The virtue in most request is conformity. Self-reliance is its aversion. It loves not realities and creators, but names and customs.

According to the text, what does society provide in return for the surrender of liberty and culture?
A. Food security
B. Names and customs.
C. Stocks
D. Manhood.

4. The following is adapted from Essays by Ralph Waldo Emerson:

We have a great deal more kindness than is ever spoken. Barring all the selfishness that chills like east winds the world, the whole human family is bathed with an element of love like a fine ether. How many persons we meet in houses, whom we scarcely speak to, whom yet we honor, and who honor us! How many we see in the street, or sit with in church, whom, though silently, we warmly rejoice to be with! The effect of the indulgence of this human affection is a certain cordial exhilaration. The emotions of benevolence and complacency which are felt toward others, are likened to the material effects of fire; so swift, or much more swift, more active, more cheering are these fine inward irradiations. From the highest degree of passionate love, to the lowest degree of good will, they make the sweetness of life.

Based on the passage, what is the general approach of each member of society to each other member of society?
A. They treat one another with chilly selfishness.
B. When they see one another in the street or church they silently ignore them.
C. They see one another in a complacent and benevolent kindness.
D. They experience passionate love for one another.

5. The following is adapted from Essays by Ralph Waldo Emerson:

Friendship requires that rare mean betwixt likeness and unlikeness, that piques each with the presence of power and of consent in the other party. Let him not cease an instant to be himself. Better be a nettle in the side of your friend, than his echo. The condition which high friendship demands is ability to do without it. That high office requires great and sublime parts. There must be very two before there can be very one. Let it be an alliance of two large formidable natures, mutually beheld, mutually feared, before yet they recognize the deep identity which beneath these disparities unites them.

Based on the passage, what must be true in order for friendship to thrive?
A. Each friend must strive to be as much like the other as possible.
B. Each friend must be great and sublime or the friendship will perish.
C. Each friend must be wholly different from the other.
D. Each friend must be true to himself and have a mutual appreciation for the other.

6. The following is an excerpt from Essays by Ralph Waldo Emerson:

A plentiful fortune is reckoned necessary, in the popular judgment, to the completion of this man of the world: and it is a material deputy which walks through the dance which the first has led. Money is not essential, but this wide affinity is, which transcends the habits of clique and caste, and makes itself felt by men of all classes. If the aristocrat is only valid in fashionable circles, and not with truckmen, he will never be a leader in fashion; and if the man of the people cannot speak on equal terms with the gentleman, so that the gentleman shall perceive that he is already really of his own order, he is not to be feared.

According to the passage, what needs to be true in order for men to be a complete man of the world?
A. He must be wealthy and able to use his fortune well.
B. He must be humble, regardless of his wealth.
C. He must be able to connect with men of all social classes.
D. He must be fashionable and a gentleman.

7. The following is an adaptation from Essays by Ralph Waldo Emerson. In it, Emerson discusses the works of William Shakespeare:

His lyric power lies in the genius of the piece. The sonnets, though their excellence is lost in the splendor of the dramas, are as inimitable as they: and it is not a merit of lines, but a total merit of the piece; like the tone of voice of some incomparable person, so is this a speech of poetic beings, and any clause as unproducible now as a whole poem. Though the speeches in the plays, and single lines, have a beauty which tempts the ear to pause on them for their euphuism, yet the sentence is so loaded with meaning, and so linked with its foregoers and followers, that the logician is satisfied. His means are as admirable as his ends; every subordinate invention, by which he helps himself to connect some irreconcilable opposites, is a poem too.

According to the text, what makes the work of Shakespeare so admirable?
A. It is part of a greater body of works by English authors.
B. Each part is beautiful on its own and as a part of a beautiful whole.
C. Shakespeare’s tone is incomparable to that of any other writer.
D. Each line is better than the one that is written before it.

8. The following is an adaptation from Essays by Ralph Waldo Emerson:

Prudence is the virtue of the senses. It is the science of appearances. It is the outermost action of the inward life. It moves matter after the laws of matter. It is content to seek health of body by complying with physical conditions, and health of mind by the laws of the intellect. The world of the senses is a world of shows; it does not exist for itself, but has a symbolic character; and a true prudence or law of shows recognizes the co-presence of other laws and knows that its own office is subaltern; knows that it is surface and not center where it works. Prudence is false when detached. It is legitimate when it is the Natural History of the soul incarnate, when it unfolds the beauty of laws within the narrow scope of the senses.

Prudence, as expressed by the passage, exists for what reason?
A. Not for itself, but to work on the surface seeking the health of the person.
B. To seek only after appearances in order to best present the body to the world.
C. To be a symbol of good moral character when interacting with the world.
D. Not for itself, but to seek good in the world around.

9. The following is an excerpt from Essays by Ralph Waldo Emerson:

The actions and events of our childhood and youth are now matters of calmest observation. They lie like fair pictures in the air. Not so with our recent actions,—with the business which we now have in hand. On this we are quite unable to speculate. Our affections as yet circulate through it. We no more feel or know it than we feel the feet, or the hand, or the brain of our body. The new deed is yet a part of life,—remains for a time immersed in our unconscious life. In some contemplative hour it detaches itself from the life like a ripe fruit, to become a thought of the mind. Instantly it is raised, transfigured; the corruptible has put on incorruptibly. Henceforth it is an object of beauty, however base its origin and neighborhood.

Based on the passage, why are recent memories so difficult to calmly observe compared to older memories?
A. Recent memories are still filled with emotions and still part of our life.
B. Older memories have had more time to process and fade to nothingness.
C. Recent memories have detached from life like an unripe fruit and cannot be eaten.
D. Older memories are far more precious than recent memories and so must be often thought of.

10. The following is an adaptation from Essays by Ralph Waldo Emerson:

All infractions of love and equity in our social relations are speedily punished. They are punished by fear. Whilst I stand in simple relations to my fellow-man, I have no displeasure in meeting him. We meet as water meets water, or as two currents of air mix, with perfect diffusion and interpenetration of nature. But as soon as there is any departure from simplicity, an attempt at halfness, or good for me that is not good for him, my neighbor feels the wrong; he shrinks from me as far as I have shrunk from him; his eyes no longer seek mine; there is war between us; there is hate in him and fear in me.

Based on the passage, what causes the relationship between two people to sour?
A. Simple relations that never progress to deeper understanding of one another.
B. Meeting as two currents of air that must mix together in unity.
C. Any complication of a simple interaction or show of selfishness.
D. A war or argument between mutual friends and neighbors.

  1. B. In the first sentences we see that men without power (farmers) think that power and place are good things while forgetting the cost that “the president” has paid for his position of power in the White House. The rest of the passage builds on this idea, explaining the costs of gaining and maintaining power. This makes answer B correct and answers C and D incorrect. Answer A is incorrect as the author does not state power will always corrupt.
  2. C. This question is answered in the first sentence where the author says that the scholar of the first age received “the world”. He then brooded thereon and gave “it” the new arrangement. From thereon “it” refers to “the world”. All other answers are incorrect readings of the context.
  3. A. In the second sentence the author contends that “society is a… company in which members agree, for the better securing of his bread… to surrender the liberty and culture of the eater.” In other words, in order to survive and have food on the table, people agree to conform to the rules of society. This make answer A correct and the others incorrect.
  4. C. The author says extensively throughout the passage that, even toward strangers, people tend to be “a great deal more kind than is ever spoken”. While Emerson does admit that there is some “selfishness that chills” he maintains that “the whole human family is bathed with an element of love like a fine ether.” This best fits with answer C. Answers A and B are too negative and answer D is incorrect because while some members of society will feel passionate love for some, they do not feel it for all members of society.
  5. D. The author maintains that friendship requires a balance between similarity and difference and holds that no one should “cease… to be himself”. He goes on to say that friends must “recognize the deep identity which beneath these disparities unites them.” In other words, each friend must be true to himself and appreciate the other, making option D correct. Option A is incorrect as the author thinks there should be some differences. Option B is incorrect as there is no discussion of them having to be great. Option C is incorrect as the friends must have a common “deep identity” to unite them.
  6. C. The first sentence tells us that most people think a fortune is necessary for the “competition of this man of the world”. He goes on to say that there are other necessary qualification and that “money is not necessary.” One of the author’s stated qualifications is an affinity which “transcends the habits of clique and caste and makes itself felt by men of all classes.” The author goes on to give examples of men who are not complete me of the world because they lack the ability to transcend class. This makes option C correct and the other options incorrect.
  7. B. The author says that “single lines have a beauty which tempts the ear to pause… yet the sentence is so loaded with meaning, and so linked with its foregoers and followers that the logician is satisfied.” In other words, he sees great beauty in individual lines yet “it is not a merit of lines, but a total merit of the piece.” Both the lines and the whole are beautiful. No other answer encompasses this concept.
  8. A. Answer option B is incorrect as the passage does not talk about appearances. Answer option C is incorrect as prudence has a symbolic character, but is not symbolic in and of itself. Answer option D is incorrect as there is no discussion of seeking good.
  9. A. We see in the passage that older memories are matters of calmest observation, but “not so with our recent actions” because “our affections as yet circulate through it.” Our affections could also be called our emotions. Later on, the author clarifies that “the new deed is yet a part of life”, giving further reason why detached observation is difficult. This makes option A the best answer. The other options are all wrong due to misreading of the passage.
  10. C. In the passage the author describes the simplicity of human interactions, but holds that “as soon as there is any departure from simplicity, an attempts at… good for me that is not good for him, my neighbor feels the wrong” and the hate ensues. Something that is attempted that is good for one’s self but not the neighbor could be described as selfishness, which makes answer C the best option. The author does not describe a relationship that is always simple as bad, making answer A wrong. The simple mixing of two currents of air is seen as the positive relationship prior to it souring, so answer B is wrong, and the war and argument in answer D must happen after the relationship has already soured. It is therefore not causal.

Digital SAT Text Structure and Purpose Reading Practice Questions #2

1. The following is an excerpt from Jane Austin’s Pride and Prejudice. In it, Mrs. Bennet seeks to get to know her new neighbor, Mr. Bingley:

An invitation to dinner was soon afterwards dispatched; and already had Mrs. Bennet planned the courses that were to do credit to her housekeeping, when an answer arrived which deferred it all. Mr. Bingley was obliged to be in town the following day, and consequently unable to accept the honor of their invitation. Mrs. Bennet was quite disconcerted. She could not imagine what business he could have in town so soon after his arrival in Hertfordshire; and she began to fear that he might always be flying about from one place to another, and never settled at Netherfield as he ought to be. What choice best describes the function of the third sentence in the overall structure of the text?

A. It explains Mrs. Bennet’s reaction to the dishes that were planned for dinner that evening.
B. It connects the description of Mr. Bingley’s actions to an explanation of his trip out of town.
C. It shows Mrs. Bennet’s feelings about her invitation to dinner being rejected.
D. It highlights the variety of emotions which Mrs. Bennet is known for.

2. The following is an excerpt from Jane Austin’s Pride and Prejudice:

The village of Longbourn was only one mile from Meryton; a most convenient distance for the young ladies, who were usually tempted thither three or four times a week, to pay their duty to their aunt, and to a milliner’s shop just over the way. The two youngest of the family, Catherine and Lydia, were particularly frequent in these attentions: their minds were more vacant than their sisters’, and when nothing better offered, a walk to Meryton was necessary to amuse their morning hours and furnish conversation for the evening; and, however bare of news the country in general might be, they always contrived to learn some from their aunt. At present, indeed, they were well supplied both with news and happiness by the recent arrival of a militia regiment in the neighborhood; it was to remain the whole winter, and Meryton was the head-quarters.

What choice best describes the function of the underlined portion in the overall structure of the text?
A. It paints a cruel picture of the abilities of Catherine and Lydia that is then qualified by a description of their meaningful trips to Meryton.
B. It addresses one reason why Catherine and Lydia so enjoy their time in Meryton which is then further explained in the subsequent sentences.
C. It shows the author’s general disregard for the shopping, gossip, and visiting that often occupy the time of young women.
D. It shows that the two girls in question have a greater capacity for learning than their sisters, as they haven’t filled their minds with the silly things that are described before and after.

3. The following is an excerpt from Jane Austin’s Pride and Prejudice:

She did at last extort from her father an acknowledgment that the horses were engaged; Jane was therefore obliged to go on horseback, and her mother attended her to the door with many cheerful prognostics of a bad day. Her hopes were answered; Jane had not been gone long before it rained hard. Her sisters were uneasy for her, but her mother was delighted. The rain continued the whole evening without intermission; Jane certainly could not come back.

What choice best describes the function of the underlined portion in the overall structure of the text?
A. It uses a seeming contradiction to foreshadow what many might see as bad luck, but what the mother sees as a very positive turn of events.
B. It shows the mother’s general lack of care for her daughters and the outcome of their day it then transitions into a description of that poor day.
C. It highlights the lack of understanding the mother has as to the weather and her daughter’s lack of safety on horseback.
D. It sets up a paradox which is further built upon in subsequent portions of the passage.

4. The following is an excerpt from Jane Austin’s Pride and Prejudice:

Elizabeth joined them again only to say that her sister was worse, and that she could not leave her. Bingley urged Mr. Jones’s being sent for immediately; while his sisters, convinced that no country advice could be of any service, recommended an express to town for one of the most eminent physicians. This she would not hear of; but she was not so unwilling to comply with their brother’s proposal; and it was settled that Mr. Jones should be sent for early in the morning, if Miss Bennet were not decidedly better. Bingley was quite uncomfortable; his sisters declared that they were miserable. They solaced their wretchedness, however, by duets after supper; while he could find no better relief to his feelings than by giving his housekeeper directions that every possible attention might be paid to the sick lady and her sister.

What choice best describes the function of the final sentence in the overall structure of the text?
A. It completes the description of the dinner party by describing the after-supper entertainment taken up by the guests.
B. It explains the two very different ways that Bingley and his sisters deal with the illness of Miss Bennet.
C. It clarifies the prior description of the search for a good physician for the sick woman.
D. It explains Mr. Bingley’s orders to the housekeeper and her skill in working to heal the young lady who is ill.

5. The following is an excerpt from Jane Austin’s Pride and Prejudice:

As a clergyman, moreover, I feel it my duty to promote and establish the blessing of peace in all families within the reach of my influence; and on these grounds I flatter myself that my present overtures of good-will are highly commendable, and that the circumstance of my being next in the entail of Longbourn estate will be kindly overlooked on your side, and not lead you to reject the offered olive branch. I cannot be otherwise than concerned at being the means of injuring your amiable daughters, and beg leave to apologize for it, as well as to assure you of my readiness to make them every possible amends.

What choice best describes the function of the first sentence in the overall structure of the text?
A. It explains the profession of the speaker so that the reader can empathize with the speaker’s situation.
B. It shows the speaker’s mission to find a way to bring peace to all families which is then followed up with a description of his plan.
C. It explains why the speaker feels it is his duty to make amends by offering up an olive branch.
D.  It introduces the idea of the speaker as a man of authority who each reader should do well to listen to.

6. The following is an excerpt from Jane Austin’s Pride and Prejudice:

In describing to her all the grandeur of Lady Catherine and her mansion, with occasional digressions in praise of his own humble abode, and the improvements it was receiving, he was happily employed until the gentlemen joined them; and he found in Mrs. Philips a very attentive listener, whose opinion of his consequence increased with what she heard, and who was resolving to retail it all among her neighbors as soon as she could. To the girls, who could not listen to their cousin, and who had nothing to do but to wish for an instrument, and examine their own indifferent imitations of china on the mantel-piece, the interval of waiting appeared very long. It was over at last, however. The gentlemen did approach.

What choice best describes the function of the underlined passage in the overall structure of the text?
A. It explains Mrs. Phillips’s plan to sell to her neighbors the china from her mantel-piece.
B. It clarifies why Mrs. Philips was willing to listen to him when his cousins were not.
C. It builds on the previous description of Lady Catherine and her home.
D. It tells the reader what Mrs. Philips is planning to do with the information from the conversation.

7. The following is an excerpt from Jane Austin’s Pride and Prejudice:

Elizabeth had heard, soon after her arrival, that Mr. Darcy was expected there in the course of a few weeks; and though there were not many of her acquaintance whom she did not prefer, his coming would furnish one comparatively new to look at in their Rosings parties, and she might be amused in seeing how hopeless Miss Bingley’s designs on him were, by his behavior to his cousin, for whom he was evidently destined by Lady Catherine, who talked of his coming with the greatest satisfaction, spoke of him in terms of the highest admiration, and seemed almost angry to find that he had already been frequently seen by Miss Lucas and herself.

What choice best describes the function of the underlined portion in the overall structure of the text?
A. To explain Elizabeth’s negative feelings toward Mr. Darcy
B. To clarify why Elizabeth was looking forward to Mr. Darcy’s arrival
C. To introduce Mr. Darcy as one of the many friends of Elizabeth
D. To explain Elizabeth’s reaction toward Mr. Darcy’s arrival

8. The following is an excerpt from Jane Austin’s Pride and Prejudice:

The whole party were in hopes of a letter from Mr. Bennet the next morning, but the post came in without bringing a single line from him. His family knew him to be, on all common occasions, a most negligent and dilatory correspondent; but at such a time they had hoped for exertion. They were forced to conclude, that he had no pleasing intelligence to send; but even of that they would have been glad to be certain.

What choice best describes the function of the underlined phrase in the overall structure of the text?
A. It gives a possible explanation for the situation described in the first sentence.
B. It introduces an aspect of Mr. Bennet’s character that is later disputed.
C. It helps the reader understand why Mr. Bennet lacks pleasing intelligence to send to his family.
D. It shows the family’s depth of knowledge of Mr. Bennet’s character which contrasts with the further characterization of their relationship.

9. The following is an excerpt from Jane Austin’s Pride and Prejudice:

Her husband allowed her to talk on without interruption while the servants remained. But when they had withdrawn, he said to her, “Mrs. Bennet, before you take any, or all of these houses, for your son and daughter, let us come to a right understanding. Into one house in this neighborhood they shall never have admittance. I will not encourage the imprudence of either, by receiving them at Longbourn.”

What choice best describes the function of the final sentence in the overall structure of the text?
A. It explains why all the homes of the neighborhood might be open to them.
B. It clarifies the earlier statement by stating which house the son and daughter will not be allowed into.
C. It completes Mr. Bennet’s explanation of the housing shortage in the neighborhood.
D. It shows Mr. Bennet’s feelings about the imprudence of his wife.

10. The following is an excerpt from Jane Austin’s Pride and Prejudice:

She began now to comprehend that he was exactly the man who, in disposition and talents, would most suit her. His understanding and temper, though unlike her own, would have answered all her wishes. It was a union that must have been to the advantage of both: by her ease and liveliness, his mind might have been softened, his manners improved; and from his judgment, information, and knowledge of the world, she must have received benefit of greater importance.

What choice best describes the function of underlined portion in the overall structure of the text?
A. It explains how each of them might benefit from a relationship with the other.
B. It clarifies the differences between the two and why she is inclined to give him a chance.
C. It explains how he might benefit from a connection to her.
D. It explains how she might benefit from a connection to him.

  1. C. In the first two sentences we learn that Mrs. Bennet had invited Mr. Bingley to dinner and had already planned the meal when Mr. Bingley responded to the invitation with a rejection, for he was to be out of town that evening. This leads to Mrs. Bennet’s reaction of being “disconcerted.” The sentence thereafter gives us further insight into her thoughts on the matter. This makes option C the best answer. Answer A is incorrect as she is disconcerted that her invitation has been refused, not at the dishes that were planned. Answer B is incorrect as the later half of the passage does not describe his trip out of town. Answer D is incorrect for while it does show Mrs. Bennet’s feelings on the matter, it does not show a variety of emotions.
  2. B. The first sentence explains that the young ladies often go to Meryton to visit their aunt and to shop. The next sentence then clarifies that Catherine and Lydia are especially likely to do so because they have vacant minds and needed constant amusement which Meryton provides. This best fits with answer option B. Their vacant minds are one reason why they so enjoy shopping and visiting in Meryton. Answer option A is incorrect as their trips to Meryton are not very meaningful. In addition, while this description seems cruel to our eyes, there is no evidence that the author means it as anything more than an accurate explanation of the characters. Option C is incorrect as there is no evidence that the author is making a general statement about the merit of these activities. Option D is incorrect as the “empty minds” are not shown to be a positive thing.
  3. A. In the underlined portion of the text, the mother is very cheerful about her predictions of a bad day. The following part of the passage clarify that the rain that the mother predicted forces Jane to stay away and “delights” the mother. This best fits with option A. The underlined text seems to be a contradiction, but is setting up the rain, a negative, being seen as a positive in the mother’s eyes. Option B is incorrect as there is no evidence that the mother does not care for her daughter, simply that for some reason she wishes her to be kept away for awhile. Answer C is incorrect as the mother understands the weather quite well and even predicts the rain. Answer D is incorrect as this is not a paradox which is a statement which contradicts itself.
  4. B. The sentence prior to the underlined part tells that Mr. Bingley is uncomfortable and his sisters miserable due to the illness of Miss Bennet. The final sentence describes their reactions to their discomfort and misery. The sisters sing duets to find solace and Mr. Bingley orders his housekeeper to give as much attention to the sick lady and her sister as possible. This best fits answer option B. Answer option A is incorrect as there is no evidence of a dinner party prior to these activities. Answer C is incorrect as it does not have anything to do with a physician (doctor). Answer option D is incorrect as that answer option ignores the sisters’ duets and falsely claims that the housekeeper is working to heal the sick lady when there is no evidence of that.
  5. C. In the underlined portion the speaker explains that he is a clergyman, or a priest. He clarifies that because of this profession he believes he must encourage peace in any family he can influence. In the rest of the passage, then he is apologizing to the listener and offering an olive branch, a well known symbol of peace. This best fits answer C. The speaker, as a clergyman, feels he must apologize and make amends for the sake of peace. Answer A is incorrect as the reader is not asked to empathize with the speaker. Answer B is incorrect as there is no plan to bring peace to all families, rather an attempt to bring peace to one. Answer D is incorrect as the reader is not admonished to listen to the speaker.
  6. D.  In the underlined portion we learn that Mrs. Philips is resolving to retail “it” among her neighbors. The “it” in question is the conversation she is having with the man and “retailing it” could also be described as passing on the information. This makes choice D the best answer. It is not choice A as she is not literally selling anything. It is not choice B, as we do not know why Mrs. Philips is listening to him. It is not choice C as it does not further describe the home in question.
  7. A. Be careful with the negatives in the underlined portion. There are two “nots” which essentially cancel out leaving us with the meaning: there were many of her acquaintances whom she did prefer. In other words, the underlined portion is explaining that Elizabeth does not prefer the company of Mr. Darcy. This best fits with answer option A. The rest of the passage explains why she might look forward to his arrival, but the underlined portion does not, making option B incorrect. Mr. Darcy is a not well like acquaintance of Elizabeth, not a friend, making option C incorrect. Mr. Darcy has not yet arrived, making option D incorrect.
  8. A. In the first sentence we learn that Mr. Bennet had not written them a letter. In the underlined portion this is explained by saying that his family is aware that is a negligent correspondent. In other words, he doesn’t do a good job of writing letters regularly. This makes option A the best answer. Option B is incorrect because the idea of Mr. Bennet being a poor correspondent is not later disputed. Option C is incorrect because it doesn’t not explain why Mr. Bennet has no good news for his family. Option D  is incorrect as there is no later contrast or further description of the relationship.
  9. B. The final statement is clarifying Mr. Bennet’s earlier declaration that there is one house into which the son and daughter will not be allowed: they are not allowed at Longbourn. This makes answer B the best option. It is not answer A as it does not explain why the children are welcome at other homes. Answer C is incorrect as they are not discussing a housing shortage. Answer D is incorrect as Mr. Bennet is discussing the imprudence of the son and daughter, not of his wife.
  10. D. Earlier in the sentence the speaker clarifies that a union between the two would benefit both. Then it explains how he would be benefited (improved manners) and finally, the underlined portion explains the benefit to her (greater importance). This makes option D correct and the others incorrect.

Practice your reading comprehension skills by reading more of Pride and Prejudice for free online:
https://www.gutenberg.org/cache/epub/1342/pg1342-images.html

Digital SAT Practice Rhetorical Synthesis #2

1. While researching a topic, a student has taken the following notes:

  • Egyptians had three kinds of paintings: one on flat surfaces, a second on bas-reliefs, and a third on designs in intaglio.
  • Intaglio designs are those hollowed out from a flat surface. Bas-reliefs are designs which are raised from the surface.
  • Egyptian painting is generally found on walls in temples and tombs as well as on columns and cornices. It is also found on small articles.
  • Egyptian painting lacks the perspective of modern art. All paintings are represented in the same plane, not painted to give the appearance of depth.
  • Painting was not signed or attributed to certain artists in ancient Egypt. For this reason, modern scholars have no knowledge of the people who created the paintings from that time period.

The student wants to educate someone unfamiliar with Egyptian paintings as to the difference between intaglio and bas-relief painting. Which choice most effectively uses relevant information from the notes to accomplish this goal?

A. Intaglio and bas-relief paintings are both forms of art that were prevalent in Ancient Egypt, and were found on walls, columns, and cornices.
B. Intaglio designs are those painted on hallowed out sections of a flat surface while bas-relief paintings are raised from the surface. Both were common in Ancient Egyptian art.
C. Modern scholars do not know the names of Ancient Egyptian painters, because their work is not signed. However, they do know that both intaglio and bas-relief painting were common.
D. In both intaglio and bas-relief painting, there is a lack of perspective that makes the art appear flat, especially to the modern eye.

2. While researching a topic, a student has taken the following notes:

  • So few fragments of Assyrian painting survive today that very little is known about the artwork of this civilization.
  • Assyrian painters appear to have done much of their work on walls, both on plaster and directly on brick.
  • When Assyrian plaster paintings are discovered, they often quickly disappear due to being exposed to the air for the first time in centuries.
  • Assyrians painted on tiles as well. Tile painting survives much longer than wall painting, but is prone to breaking. No complete Assyrian tile paintings have been assembled.

The student wants to give an overview of Assyrian painting to an audience who has no knowledge of it. Which choice most effectively uses relevant information from the notes to accomplish this goal?

A. While little is known about Assyrian painting, it seems that it was largely done on walls and on tiles.
B. Unearthed Assyrian wall painting often degrades quickly when exposed to the air, which makes it difficult to learn about this style of art.
C. Very little Assyrian painting survives today, so art historians know nothing about the art of this civilization.
D. Painted Assyrian tiles, or shards thereof, are often found. However, archeologists have been unable to assemble entire paintings.

3. While researching a topic, a student has taken the following notes:

  • In  6th century B.C.E. Babylon under king Nebuchanezzar, tile-painting was the peak of artistic expression.
  • The Babylonians were known for their rich and glowing cities, which shone with the bright and vibrant colored tiles.
  • The Babylonian tiles could be a solid color or could have intricate patterns on them. Some tiles were used to assemble vast scenes similar to those found on medieval tapestries.
  • Only a few remnants of Babylonian tile painting still exist today.

The student wants to explain to readers the beauty of Babylonian tile painting. Which choice most effectively uses relevant information from the notes to accomplish this goal?

A. Babylonian cities were rich and glowing, covered in bright and vibrantly colored tiles which made up intricate designs, some even as complex as medieval tapestries.
B. Under King Nebuchanezzar, tile-painting was the peak of artistic expression, however, few examples remain.
C. Babylonian tiles were often a solid color, though many could be covered with a painted pattern.
D. Painted tiles covered the Babylonian cities of the 6th century B.C. E. The cities were, in fact, known for their colored tiles.

4. While researching a topic, a student has taken the following notes.

  • Apelles could be described as the greatest of all Greek painters.
  • Apelles’s art is known for exquisite finishing as well as an abundance of grace and sweetness.
  • Apelles painted both King Phillip of Macedon and his son: Alexander the Great. This work lead to Apelles being the court painter and granted the exclusive rights to paint the monarch.
  • Late in his career, Apelles traveled as far as Egypt and painted a diverse range of mythological beings including Venus.

The student wants to include in her essay a sentence that will highlight Apelles’s connections to the Greek court. Which choice most effectively uses relevant information from the notes to accomplish this goal?

A. The greatest of all Greek painters, Apelles was well known at court for the grace and sweetness in his work.
B. While he was well known in Greece, Apelles also traveled widely to seek inspiration for his work
C. Apelles painted a wide variety of subjects, from Phillip of Macedon, to mythological beings like Venus.
D. Both Phillip of Macedon and his son, Alexander the Great, retained Apelles as their official portrait painter and the general court painter as well.

5. While researching a topic, a student has taken the following notes:

  • One well known form of Roman art is mosaics, which were used not just as artistic pieces, but also as wall and floor decorations.
  • The Romans did not invent the mosaic art form, but they did develop their own unique style of placing thousands of tiny shards of stone, glass, and pottery to create pictures.
  • Romans spread their mosaic art so far and wide that it is still common today for excavations in far flung parts of the former Roman empire to find amazing pieces of art buried in fields and under buildings.
  • Due to the prolific nature of Roman mosaics, many mosaics that are discovered are not preserved.

The student would like to explain to a friend already familiar with mosaics why Roman mosaics often are not preserved after discovery. Which choice most effectively uses relevant information from the notes to accomplish this goal?

A. Roman mosaics were used as wall and floor decorations and were made up of shards of glass, stone, or pottery, which made them quite durable.
B. Mosaics are still commonly found in far flung parts of the former Roman empire as the durability of the materials has allowed them to survive significantly longer than other art forms.
C. Due to the widespread nature of Roman mosaics, newly unearthed mosaics are often not preserved: there are simply too many to keep them all.
D. Mosaic tiling was known prior to the Romans adopting the art form, but the Romans perfected and spread the art form.

6. While researching a topic, a student has taken the following notes:

  • Roman catacomb paintings from the 3rd and 4th centuries C.E. often have Christian themes and are found in the underground catacombs near Rome itself since early Christians often had to hide their religious activities for fear of persecution.
  • These paintings exhibit majesty and earnestness as the painters tried to express their faith through their artwork.
  • Many of these artworks had not just literal meanings, but also symbolic meanings that would only be apparent to others who understood the significance of the chosen symbols.
  • One example of this symbolism would be a literal painting of a river of water which also had a figurative meaning in regards to baptism: a spiritual washing away of sin.

The student would like to explain the symbolism often found in catacomb paintings. Which choice most effectively uses relevant information from the notes to accomplish this goal?

A. Catacomb paintings in Rome often have both obvious and symbolic themes. The obvious would be apparent to any viewer, while the symbolic would only be known to those who could decipher them.
B. Christian symbols are often found in catacomb paintings as the painters strove to express their faith through art and still avoid persecution. One example of such a symbol would be an artist painting flowing water to symbolize baptism.
C. Known for their majesty and earnestness, many early Christian catacomb paintings from the 3rd and 4th centuries B.C.E. are still visible in Rome today.
D. Due to fear of religious persecution, many early Christian artists used symbols in their paintings and in their every day life.

7. While researching a topic, a student has taken the following notes:

  •  During the Romanesque period (950-1250) painting was not seen as an elevated art form. Rather, it was looked at as a form of visual story telling.
  • Most painting in Europe was done to illustrate religious stories and concepts and so was under the control and direction of the church.
  • Because of this connection, glass painting (what we know today as stained-glass) was developed, though historians argue whether the technique originated in Germany or France.
  • While most glass painting was very colorful as we are accustomed to seeing today, some factions of the church rejected highly colorful paintings and instead opted for grisaille glass which utilized the same patterns, but only applied paints in shades of grey, green, and brown with only very small bursts of other colors.

The student wants to emphasize the difference in how painting was seen in the Romanesque Period compared to how we see it today. Which choice most effectively uses relevant information from the notes to accomplish this goal?

A. During the Romanesque period, painting was primarily done on glass in order to illustrate religious concepts.
B. Today, painting is seen as a form of high art, while in the Romanesque period, it was used merely as a way to tell stories.
C. During the Romanesque period, most painting was done in bright and vibrant colors, however, some religious orders preferred to paint mostly in shades of grey, green, and brown.
D. Stained glass art in the Romanesque period was seen as a way to tell the stories of the church. Today, it is also used in a wide variety of non-religious settings.

8. While researching a topic, a student has taken the following notes:

  • The Gothic Period of Medieval painting was highly influenced by the well known Gothic style of architecture.
  • In the Gothic Period, painting moved out from the influence of the clergy and painters were more and more free to paint how and what they wanted, backed by powerful guild associations.
  • Gothic painting includes a lot of images from nature and starts to attempt to introduce perspective (which had previously been missing) into artistic renderings of scenes.
  • Due to the highly decorative nature of Gothic architecture, not much space was left on walls for painting. Glass painting, therefore, remained very common.

The student wants to explain the change during the Gothic period and the results of that change. Which choice most effectively uses relevant information from the notes to accomplish this goal?

A. Influenced by Gothic architecture, Gothic painting was both highly decorative, and often created on windows.
B. Both Gothic architecture and a lack of clerical oversight influenced a change in painting arts during the Gothic period.
C. While art had previously been quite flat, Gothic painters started to experiment with adding dimension and perspective to their paintings.
D. During the Gothic period, the church lost much influence over artists. This led to paintings incorporating perspective and natural themes.

9. While researching a topic, a student has taken the following notes:

  • Giotto was a well known artist in Italy  in the late 13th and early 14th century.
  • Giotto’s earliest known work is a portrait of the author Dante, painted on the wall of the Podesta at Florence.
  • Dante was exiled from Florence and his portrait was whitewashed over, which preserved it for modern scholars to examine.
  • There are many other works of Giotto in Florence and is also known for his frescoes in Assisi, and his pictures at Padua
  • In addition to his painting, Giotto is remembered as the great architect who designed the tower in Florence.

The student wants to give an overview of Giotto’s work in Florence. Which choice most effectively uses relevant information from the notes to accomplish this work?

A. Giotto’s first work is found on a wall in Florence. It is a painting of the author Dante. In addition to his paintings, Giotto is also known for his frescoes in Assisi and his pictures at Padua.
B. The painting of Dante at the Podesta is the first known work of Giotto and was preserved when it was whitewashed over following Giotto’s exile from Florence.
C. Giotto’s first work, a painting of Dante, as well of several other paintings still exist in Florence. In addition, he is remembered as the architect of the tower in Florence.
D. Despite being exiled from Florence, Dante is still associated with the city because of his work designing the tower in Florence.

10. While researching a topic, a student has taken the following notes:

  • The Van Eyck family, of Flemish origin, is well known for its artistic creations.
  • The family had four painters: three brothers and one sister, the eldest of which was Hubert Van Eyck.
  • Hubert Van Eyck work and discoveries on the use of colors led to modern day oil painting. After Hubert passed away, his brother, Jan, perfected the technique and became a celebrated painter.
  • Prior to the work of the Van Eyck brothers, oil painting was a very unsatisfactory process and led to mixed results. The Van Eyck method vastly improved the process and results of oil painting.

The student wants to explain to a friend the influence the Van Eyck family had on oil painting. Which choice most effectively uses relevant information from the notes to accomplish this goal?

A. There were four painters in the Van Eyck family. Two of them, Hubert and Jan, are the best remembered of the family.
B. Huber and Jan Van Eyck worked to perfect the process of oil painting and discovered techniques which led to modern oil painting.
C. Traditional oil painting was a very unpredictable process which caused issues for most painters, including all four of the Van Eyck siblings.
D. The Va Eyck family is a well known painting family. The three brothers and one sister are well known for their artistic masterpieces.

Answer Explanations:

  1. B. The question is asking for the difference between intaglio and bas-relief painting. Options A and D give the similarities between the two and are therefore incorrect. Option C focuses on the artists, not the art and only gives a similarity between the two approaches. Option B takes information from the second bullet point to point out the differences between the two approaches and is therefore correct.
  2. A. Given that the audience has no knowledge of Assyrian painting, the student must give even the most basic information. Option A best gives a basic understanding of Assyrian painting based on the notes. Option B explains why we struggle to understand Assyrian art, but does not give an overview of what we do know. Option C explains why we don’t know much but doesn’t explain what we do know. Option D is not an overview of Assyrian painting, but rather a fact about only one type of Assyrian painting.
  3. A. The student wants to explain the beauty of the Babylonian tile painting. Options B, C and D explain different facts about Babylonian tiles and painting, but do not explain their beauty. Option A explains their brightness and vibrancy, their intricate designs, and even compares them to tapestries, giving a clear understanding of the beauty of this art form.
  4. D. The student wants to talk not about Appelles’s work, but rather his connection to the Greek court (the organization surrounding the Greek monarch). The answer, therefore, must focus on Apelles’s connections to the kings Phillip of Macedon and Alexander the Great. This eliminates options A and B. Option C mentions Phillip of Macedon, but only as a subject of a painting. Option D best explains Apelles’s close connection to the court of Phillip and Alexander.
  5. C. The student’s friend is already familiar with mosaics in general, so the concept does not need to be explained, eliminating option A. The student wants to explain why Roman mosaics, once discovered, are often not preserved. Options B and D do not explain this, but rather give other details about the mosaics. Option C explains that there are simply too many mosaics to preserve them all, giving the explanation the student wants.
  6. B. The student wants to explain the symbolism found specifically in catacomb paintings. Option A talks about the existence of symbols, but does not give an explanation of them. Option D does not discuss catacomb paintings specifically. Option C does not address symbols at all. Option B, therefore, is the best answer since it addresses catacomb painting specifically, addresses why they were used, and gives an explanation of one specific symbol.
  7. B. The student wants to emphasize the difference between how we see art today (as an elevated art form) and how people saw it in the Romanesque period (as a means of visual storytelling). Only option B provides this contrast. Options  A and C speak to Romanesque painting, but not to modern painting. Option B does not discuss how the painting is seen, but rather how stained glass is used.
  8. D. The correct answer must include both the change during the Gothic period and the results of that change. Option A does not discuss how Gothic painting changed. Option B discusses why a change occurred, but not what that change was. Option C discusses a change, but not the effects thereof. Option D is the best answer since it discusses a change (the church lost influence) and the effects thereof (artists started using perspective and natural themes).
  9. C.  The correct answer must give a broad overview of Giotto’s work specifically in Florance. Option A speaks of his work in multiple places. Options B and C speak of just one Florentine work of Giotto. Option C gives the best general overview of Giotto’s work in Florence.
  10. B. In order to understand the influence the family as a whole had on oil painting, option B is the best answer since it discusses the influences of multiple members of the family. Option A says they are remembered, but not any influence that they had. Option C discusses an issue they had. D presents the idea that they are well known, but not that they had any influence on oil painting in general.

The information for all notes is taken from History of Art for Beginners and Students. If you wish to read more it is available in the public domain from Project Gutenberg:

https://www.gutenberg.org/cache/epub/24726/pg24726-images.html#CHAPTER_I

Digital SAT Standard English Conventions Practice Questions: Punctuation

1.  The hour of noon had just struck, and the few visitors still lingering among the curiosities of the great museum were suddenly startled by the sight of one of the attendants running down the broad, central staircase, loudly shouting, “Close the doors! Let no one out! An accident has ____________________ to leave the building.”

Which choice completes the text so that it conforms to the conventions of Standard English?
A. occurred, while nobody’s
B. occurred, while nobodies
C. occurred, and nobody is
D. occurred, and nobodies

2. She did not answer. She did not even look his way. With a rapid glance into the faces __________________ in one of deep compassion directed toward herself, he repeated his question.

Which choice completes the text so that it conforms to the conventions of Standard English?
A. about him, ending
B. about him; ending
C. about him ending
D. about him ending,

3. With an air of relief Mr. Jewett stepped again into the court and, repelling with hasty gestures the importunities of the small group of men and women who had lacked the courage to follow the more adventurous ______________________ to where the door-man stood on guard over the main entrance.

Which choice completes the text so that it conforms to the conventions of Standard English?
A. ones upstairs—crossed
B. ones upstairs crossed
C. ones upstairs crossed,
D. ones upstairs, crossed

4. They were standing at the foot of the great staircase connecting the two floors. __________________ away on either side, ran the two famous, highly ornamented galleries, with their row of long low arches indicating the five compartments into which they were severally divided.

Which choice completes the text so that it conforms to the conventions of Standard English?
A. Above them, stretching
B. Above, them stretching
C. Above them stretching
D. Above them—stretching

5. The Curator offered his arm. The old man made a move to ___________________ himself up with an air of quiet confidence.

Which choice completes the text so that it conforms to the conventions of Standard English?
A. take it then, drew
B. take it then drew
C. take it—then drew
D. take it: then drew

6. She was near ______________________ being a woman of great nerve, she fought her weakness and waited patiently for the next question. It was different, without doubt, from any she had expected.

Which choice completes the text so that it conforms to the conventions of Standard English?
A. collapsing again, however,
B. collapsing again; however,
C. collapsing again: however,
D. collapsing again, however;

7. As he was meditating how he could best convey to her the necessity of detaining her further, he heard a muttered exclamation from the young woman standing near her and, following the direction of her pointing finger, saw that the strange silence which had fallen upon the room had a _______________ had fainted away in her chair.

Which choice completes the text so that it conforms to the conventions of Standard English?
A. cause; Mrs. Taylor,
B. cause: Mrs. Taylor
C. cause, Mrs. Taylor
D. cause Mrs. Taylor,

8. But the detective was not so hasty. With a thousand things in mind, he stopped to peer along the gallery and down into the court before giving himself away to any prying eye. Satisfied that he might make the desired move with impunity, Mr. Gryce was about to turn in the desired direction when, struck by a ______________ again stopped short.

Which choice completes the text so that it conforms to the conventions of Standard English?
A. new fact: he
B. new fact, he,
C. new fact he
D. new fact, he

9. The detective, thus appealed to, hesitated a ____________________ an irrelevance perhaps natural to the occasion, he inquired where this door so conveniently hidden from the general view led to.

Which choice completes the text so that it conforms to the conventions of Standard English?
A. a moment then; with
B. a moment; then, with
C. a moment, then, with
D. a moment then with

10. The detective, working his way back around the pedestal, cast another glance up and down the _________________ into the court. Still no spying eye, save that of the officer opposite.

Which choice completes the text so that it conforms to the conventions of Standard English?
A. gallery and over
B. gallery, and over
C. gallery and, over
D. gallery: and over

11. Silence. Heads moving, eyes peering, excitement visible in every face, but not a word from anybody. Mr. Gryce turned and pointed up at the clock. ­­­­­­­­­­­­­­­­_________________ still no word from man or woman.

Which choice completest the text so that it conforms to the conventions of Standard English?
A. All looked however
B. All looked: but
C. All looked, or
D. All looked—but

12. He decided upon the northern one, which you will remember was the one holding _________________ __ finding anybody there, no matter whom, would certainly settle the identity of the person responsible for that flying arrow.

Which choice completes the text so that it conforms to the conventions of Standard English?
A. the tapestry; since
B. the tapestry, since,
C. the tapestry since,
D. the tapestry: since,

13. A man was there: man going down—_________________ this man, as he soon saw from his face and uniform, was Correy the attendant.

Which choice completes the text so that it conforms to the conventions of Standard English?
A. no—coming up—
B. no, coming up;
C. no coming up;
D. no, coming up,

14. As he did this, two ____________________ Sweetwater, who had stolen upon the scene, possibly at some intimation from Mr. Gryce, took a step toward them which brought him in alignment with the Englishman, of whose height in comparison with his own he seemed to take careful note; and secondly, the sensitive skin of the foreigner flushed red again as he noticed the Coroner’s sarcastic smile, and heard his dry remark.

Which choice completes the text so that it conforms to the conventions of Standard English?
A. things happened; first
B. things happened; first,
C. things happened: first,
D. things happened: first

15. But nowhere in the great city of which we write on this night of May 23, 1913, was there to be found a scene of greater ______________________ the court and galleries of its famous museum.

Which choice completes the text so that it conforms to the conventions of Standard English?
A. contradictions: than in
B. contradictions than, in
C. contradictions than in
D. contradictions than in,

16. Gems of antique art, casts in which genius had stored its soul and caused to live before us the story of the _________________ from desert sands, friezes from the Parthenon and bas-reliefs from Nineveh and Heliopolis, filled every corner, commanding the eye to satisfy itself in forms of deathless grace or superhuman power.

Which choice completes the text so that it conforms to the conventions of Standard English?
A. ancients pillars
B. ancients, pillars
C. ancients pillars,
D. ancients: pillars

17. The Inspector, finding himself very much disturbed by the doubt just mentioned, felt inclined to _________________ any perceptible advancement had been made by this freak business of his canny subordinate.

Which choice completes the text so that it conforms to the conventions of Standard English?
A. question: whether
B. question whether,
C. question; whether
D. question whether

18. In saying this, Mr. Gryce studiously avoided the _____________________ the Inspector in his turn looked up, then down—anywhere but in the detective’s direction.

Which choice completes the text so that it conforms to the conventions of Standard English?
A. Inspectors’ eye;
B. Inspector’s eye;
C. Inspector’s eye:
D. Inspectors’ eye,

19. For the next three days the impatience of the public met with nothing but disappointment. The police were reticent—more reticent far than __________________________ to add to the facts already published, had little but conjectures to offer.

Which choice completes the text so that it conforms to the conventions of Standard English?
A. usual—and the papers, powerless
B. usual, and the papers, powerless
C. usual—and the papers—powerless
D. usual, and the papers—powerless

20. The other—Carleton Roberts, his bosom friend, and the museum’s chief director—is of a different _____________ no less striking to the eye.

Which choice completes the text so that it conforms to the conventions of Standard English?
A. type: but
B. type, nor
C. type, but
D. type, for

  1. C. “An accident has occurred” is an independent clause. “Nobody is to leave the building” is also an independent clause. Two independent clauses can be connected with a comma and one of the FANBOYS. “While” is not one of the FANBOYS, so A and B are incorrect. “Nobodies” is the plural form of “nobody” so D is incorrect. Answer C is correct because it correctly uses comma FANBOYS and it has the correct format of the word “nobody”.
  2. A. The phrase “ending in one of deep compassion directed toward herself” is extra information in the sentence that is not crucial to the structure of the sentence. Such phrases should be surrounded by commas, dashes, or parentheses. Since there is a comma after the word “herself” there must therefore also be a comma before the word “ending” to correctly bracket the phrase with commas. This makes A the best answer and the other answers incorrect.
  3. D. The phrase starting with the word “repelling” and ending with the word “upstairs” is extra information in the sentence that is not crucial to the structure of the sentence. Such phrases should be surrounded by commas, dashes, or parentheses. Since there is a comma before the word “repelling” there must therefore also be a comma after the word “upstairs” to correctly bracket the phrase with commas. This makes option D the best answer and the other options incorrect.
  4. A. The phrase starting with the word “stretching” and ending with the word “side” is extra information in the sentence that is not crucial to the structure of the sentence. Such phrases should be surrounded by commas, dashes, or parentheses. Since there is a comma after the word “side” there must therefore also be a comma before the word “stretching” to correctly bracket the phrase with commas. This makes option A the best answer and the other options incorrect.
  5. C. The first clause in the second sentence is an independent clause. The second clause is dependent. To connect an independent and dependent clause you should use a comma, however, this is not an option. A dash can replace a comma, giving a heavier pause. This makes answer C the best answer. Answer A puts a comma in the wrong place, answer B does not give punctuation where a pause is needed, answer D is incorrect as the second part of the sentence is not clarifying or explaining the first part.
  6. B. In this situation, the word “however” logically belongs with the second independent clause. The semicolon connecting the two independent clauses, therefore, must go after the word “again”. The word “however” is then an introductory word to the second independent clause, making it logical to place a comma thereafter. This makes option B correct and the other options incorrect.
  7. B. Options C and D are both run on sentences in which two independent clauses are incorrectly connected with a comma. The second clause is clarifying or explaining something from the first clause, namely, the reason the silence has fallen on the room. This makes a colon the best possible answer since a colon connects an independent clause to a clarification or explanation of that clause.
  8. D. The phrase “struck by a new fact” is extra information in the sentence that is not crucial to the structure of the sentence. Such phrases should be surrounded by commas, dashes, or parentheses. Since there is a comma after the word “when” there must therefore also be a comma before the word “again” to correctly bracket the phrase with commas. This makes D the best answer and the other answers incorrect.
  9. B. This sentence is really made up of two sentences. The first one ends after the word “moment”. The second sentence is made up of a dependent clause connected to an independent clause with the comma after the word “occasion”. To connect two sentences, we need to put a period or a semicolon between them. This makes option B the best answer.
  10. A. In this portion of the sentence no clauses are ending or beginning, no extra information exists, and no lists are being enumerated. In other words, there is no reason to put any sort of punctuation. This makes answer A the best option. Answer B would be correct only if the “and” were connecting two independent clauses.
  11. D. In this sentence, there is a contrast between all looking, and no one saying anything. This makes answers A and C incorrect. In addition, the second part of the sentence is not clarifying or explaining the first part, meaning that option B is incorrect. This just leaves answer D which uses the versatile dash to connect two clauses.
  12. A. The first part of this passage is a sentence made up of a dependent clause connected to an independent clause with a comma after the word “one”. The second part of the passage is an independent clause interrupted by the extra information “no matter whom”. To connect these two complete sentences, we need to use a semicolon. This makes option A correct and the other options incorrect.
  13. B. The first part of this passage is a sentence made up of an independent clause with a colon and then clarifying information ending after the word “up.” The second part of the passage is an independent clause interrupted by the extra information “as he soon saw from his face and uniform.” In order to connect these two sentences, we need to use a semicolon. This eliminates option D. The confusion of having two dashes around essential information eliminates option A. The need for a pause after “no” leads us to B as the best answer.
  14. C. A colon goes after an independent clause and before a clarification or explanation of that independent clause. A comma goes after an introductory word or phrase like “first”. This makes answer option C the best choice.
  15. C. In this portion of the sentence no clauses are ending or beginning, no extra information exists, and no lists are being enumerated. In other words, there is no reason to put any sort of punctuation.
  16. B. This sentence starts with a list. Among the items in the list are “casts in which genius had stored its soul and caused to live before us the story of the ancients” and “pillars from desert sands”. These two items must be separated with a comma, making option B the correct answer.
  17. D. In this portion of the sentence no clauses are ending or beginning, no extra information exists, and no lists are being enumerated. In other words, there is no reason to put any sort of punctuation.
  18. B. In this sentence there is a single inspector who has possession over his eye. This means that the apostrophe must go before the “s”, making options A and D incorrect. In addition, we are connecting two independent clauses, making the semicolon the best option.
  19. A. “More reticent far than usual” is extra information in the sentence. Since it begins with a dash before the word “more” we must end it with a dash after the word “usual”. The phrase “powerless to add to the facts already published” is also extra information. Since it ends with a comma after the word “published” we must start it with a comma before the word “powerless”. This makes option A the correct answer.
  20. C. The first clause of the sentence says that Carleton Roberts is a different type, this contrasts with him being “no less striking”. The contrasting word “but” therefore fits the author’s meaning. A comma to connect the independent clause to a dependent one would also be appropriate, making option C the best answer.

All passages are adapted or taken from The Mystery of the Hasty Arrow by Anna Katharine Green. You can read who whole exciting mystery novel about a murder in a museum for free online:

https://www.gutenberg.org/cache/epub/17763/pg17763-images.html